Jump to content

phudinhgioihan's Content

There have been 329 items by phudinhgioihan (Search limited from 01-06-2020)



Sort by                Order  

#540805 Tìm ma trận giao hoán

Posted by phudinhgioihan on 14-01-2015 - 17:44 in Đại số tuyến tính, Hình học giải tích

Bài 2. Cho ma trận $A=(1\;\; 0\;\; 1, 0\;\; 1\;\; 2, 0\;\; 0\;\; 1)$. Tìm ma trận vuông cấp 3 B sao cho $AB+BA=0$.

 

 

Bài 2:

 

p/s:Không biết bài này có ngụ gì hay tổng quát gì không @@

 

Tổng quát gì thì hãy để ý 2 cột đầu tiên của $A$ có gì đặc biệt? Sau đó xem tiếp bài giải:

 

Giả sử $B=[b_1,b_2,b_3]$ với $b_i \in \mathbb{R}^3 \;, i=1,2,3$

 

Ta có: $AB\begin{bmatrix}1 \\0 \\0 \end{bmatrix}+BA\begin{bmatrix}1 \\0 \\0 \end{bmatrix}=0$

$\Leftrightarrow Ab_1+b_1=0 \Leftrightarrow Ab_1=-b_1$

 

Dễ thấy $A$ chỉ có một giá trị riêng là 1, do đó phải có $b_1=0$ vì nếu $b_1 \neq 0$ thì $-1$ là trị riêng của $A$.

 

Tương tự, $AB\begin{bmatrix}0 \\1 \\0 \end{bmatrix}+BA\begin{bmatrix}0 \\1 \\0 \end{bmatrix}=0$

$\Leftrightarrow Ab_2+b_2=0 \Leftrightarrow Ab_2=-b_2 \Leftrightarrow b_2=0$

 

$A$ có một vecto riêng là $\begin{bmatrix}1 \\1 \\0 \end{bmatrix}$, ta sẽ sử dụng vecto này.

 

 $AB\begin{bmatrix}1 \\1 \\0 \end{bmatrix}+BA\begin{bmatrix}1 \\1 \\0 \end{bmatrix}=0$

$\Leftrightarrow AB\begin{bmatrix}1 \\1 \\0 \end{bmatrix}+B\begin{bmatrix}1 \\1 \\0 \end{bmatrix}=0$

$\Leftrightarrow AB\begin{bmatrix}1 \\1 \\0 \end{bmatrix}=-B\begin{bmatrix}1 \\1 \\0 \end{bmatrix}$

$\Leftrightarrow B\begin{bmatrix}1 \\1 \\0 \end{bmatrix}=0$

$\Leftrightarrow b_3=0$

 

Vậy $B=0$




#540761 $A\in M_{3}(\mathbb{R}), A\not =I_3 ,...

Posted by phudinhgioihan on 13-01-2015 - 23:39 in Đại số tuyến tính, Hình học giải tích

$$A\left ( AB-I \right )=0 \Rightarrow Im \left ( AB-I \right ) \subset \ker A \Rightarrow rank \left ( AB-I \right ) \leq n-rank A$$

Do $rank A = rank B$ và $rank \left ( AB-I \right )=rank \left ( BA-I \right )$ nên $rank \left ( BA-I \right ) \leq n- rank B$. Suy ra $Im \left ( BA-I \right ) \subset \ker B \Rightarrow B\left ( AB-I \right )=0$.

Bài giải sai rồi, $rank \left ( BA-I \right ) \leq n- rank B \Leftrightarrow dim\; Im(BA-I) \le dim \; Ker(B)$ nhưng không thể suy ra $Im(BA-I) \subset Ker B$ được.

 

Thực sự thì mình vẫn chưa hiểu cách cm lắm, chúng ta có $rank A=rank B=r$ thì $rank (AB-I)=r$ à cậu? 

 

Tiếc là sai rồi, lấy r=0 là thấy ngay.




#540755 Chứng minh A chéo hoá được

Posted by phudinhgioihan on 13-01-2015 - 22:59 in Đại số tuyến tính, Hình học giải tích

Một bài khá khá nè, mọi người làm cho vui. Có nhiều cách giải nhé.

Cho A là ma trận thoả mãn $A^3=A$, chứng minh rằng A chéo hoá được. Trường hợp $A^4=A$ thì sao?

 

Giả sử $A$ lấy hệ số trên trường $\mathbb{K}$

 

Giải một cách sơ cấp:

 

Ta có: $\frac{1}{2}x(x+1)+\frac{1}{2}x(x-1)-(x-1)(x+1)=1 \;,  \forall x \in \mathbb{K}$

 

$\Rightarrow \frac{1}{2}A(A+I)X+\frac{1}{2}A(A-I)X-(A-I)(A+I)X=X ,\; \forall X \in \mathbb{K}^n $

 

Do $A(A-I)(A+I)=0$ nên $A(A+I)X \in Ker(A-I) , \; A(A-I)X \in Ker (A+I), \; (A-I)(A+I)X \in Ker A \;, \forall X \in \mathbb{K}^n$

 

Suy ra $\mathbb{K}^n \subset Ker(A)+Ker(A-I)+Ker(A+I)$ , đồng thời $Ker(A)+Ker(A-I)+Ker(A+I) \subset \mathbb{K}^n$

nên $\mathbb{K}^n = Ker(A)+Ker(A-I)+Ker(A+I)$

 

Ký hiệu $E(\lambda)$ là không gian con riêng của $A$ ứng với giá trị riêng $\lambda$

 

Do các trị riêng của $A$ đều là nghiệm của phương trình $x^3-x=0$ nên $Spec(A) \subset \{0;-1;1\}$, hơn nữa nếu $\lambda \not\in Spec(A)$ thì $Ker(A-\lambda I) =\{0\}$ nên

 

 $$\sum_{\lambda \in Spec(A)} E(\lambda)=Ker(A)+Ker(A-I)+Ker(A+I)=\mathbb{K}^n$$

 

Vậy $A$ chéo hóa được trên $\mathbb{K}$

 

 

Đa thức cực tiểu là ước của $x^3-x$. Đa thức này chỉ có nghiệm đơn. Do đó $A$ chéo hoá được.

 

Tổng quát: Ma trận $A \in M_n(\mathbb{K})$ chéo hóa được trên $\mathbb{K}$ khi và chỉ khi tồn tại đa thức trên $\mathbb{K}[X]$ triệt tiêu A và tách đơn trên $\mathbb{K}$ (các nghiệm đều đơn và thuộc $\mathbb{K}$.

 

Chứng minh: Xem trang 67, Đại số 2, Jean Marie Monier.

 

File pdf: http://diendantoanho...n-marie-monier/

 

 

Định lý tương tự là: A chéo hóa được trên K khi và chỉ khi đa thức tối tiểu tách đơn trên K.

 

Nói thêm, các trị riêng của A là các nghiệm của đa thức tối tiểu.




#540747 Về tam giác hóa ma trận

Posted by phudinhgioihan on 13-01-2015 - 22:27 in Đại số tuyến tính, Hình học giải tích

quyển ý lai cả Việt lẫn nước ngoài mà anh , ví dụ kí hiệu $KGCR(f,\lambda)$ . Trước h em đọc mỗi quyển giáo trình của trường ( đương nhiên là ko có phần nâng ma trận chéo hóa ) và quyển của Lê Tuấn Hoa thôi ạ @@, biết thế ngày trước học quyển này.

 

Mỗi một quyển sách có một cái hay và phần mạnh riêng cho nên ráng đọc nhiều cuốn cho mỗi môn thì càng tốt em. Giáo trình của trường thú thật là coi cho biết sẽ học những gì ở lớp rồi quăng ở xó nào đó bởi nội dung trong ấy thường sơ sài. Quyển của Lê Tuấn Hoa chủ yếu được phần bài tập, mà phần này thì ông ấy lấy trong các quyển khác, phần lớn là quyển Problems in Linear Algebra của Proskuryakov. Em xem thêm quyển Problems in Linear Algebra của Prasolov nữa thì ngon.




#540719 Về tam giác hóa ma trận

Posted by phudinhgioihan on 13-01-2015 - 21:17 in Đại số tuyến tính, Hình học giải tích

nản quá, quyển này em cũng có nhưng nhiều kí hiệu lạ hoắc, thôi để thi xong học lại từ đầu =,=

Ai bảo em không chịu đọc nó rồi lại than nhiều ký hiệu lạ hoắc :P. Các ký hiệu trong đó anh thấy đều chuẩn cả, chỉ có nhiều ký hiệu trong sách Việt mới lạ thôi.




#540698 Về tam giác hóa ma trận

Posted by phudinhgioihan on 13-01-2015 - 19:26 in Đại số tuyến tính, Hình học giải tích

 Tất nhiên mình biết như vậy vì lời giải nó viết vậy mà. Nhưng chỉ có điều mình chưa từng được đọc chứng minh nào cho cái này.

 

Chứng minh tam giác hóa đồng thời và chéo hóa đồng thời là tương tự nhau. Bạn có thể xem bài tập 2.4.12 (chéo hóa đồng thời) và bài tập 3.1.12 (tam giác hóa đồng thời) trong quyển Đại số 2, Jean Marie Monier, đằng sau sách có giải chi tiết.

 

File pdf:

http://diendantoanho...n-marie-monier/




#540619 Chứng minh $r(A)+r(E-A)=n$ với $A^2=A$

Posted by phudinhgioihan on 13-01-2015 - 02:36 in Đại số tuyến tính, Hình học giải tích

Cho $A$ là ma trận vuông cấp $n$ sao cho $A^2=A$.

Chứng minh $r(A)+r(E-A)=n$

 

 

Sử dụng BDT sylvester $n =r(E)=r (A+E-A) \le r(A)+r(E-A) \le n+r[A(E-A)]=n$ suy ra đpcm

 

Sử dụng các bất đẳng thức phụ thì phải chứng minh cho nên cũng không phải cách hay. Ta thử cách giải bình thường xem :)

 

Giả sử $A \in M_n(\mathbb{R})$

 

$\forall X \in \mathbb{R}^n, \; X=AX+(E-A)X \Rightarrow \mathbb{R}^n \subset ImA+Im(E-A)$, hơn nữa, vì $Im A+Im (E-A) \subset \mathbb{R}^n$ nên

$\mathbb{R}^n=ImA+Im(E-A)$

 

Theo giả thiết suy ra $A(E-A)=0$ nên $ImA \cap Im(E-A)= \{0\}$

 

Như vậy $\mathbb{R}^n=ImA\oplus Im(E-A)$, suy ra $rank A+rank (E-A)=n$

 

Nói thêm, lời giải trên tiến thêm chút xíu là cm được $A$ chéo hóa được trên $\mathbb{R}$




#540617 Chứng minh $rank(A+B)\leq rankA + rankB$

Posted by phudinhgioihan on 13-01-2015 - 02:14 in Đại số tuyến tính, Hình học giải tích

Cho $A,B$ thuộc $M(m \times n , K)$ trong đó $K$ là một trường .

Chứng minh :

a) $rank (A+B) \leq rank A+rank B$

b) Nếu $m=n$ thì $rankA + rankB - n \leq rank(AB) \leq min(rankA,rankB)$

 

 


b, Khi $m=n$ thì đó chính là bất đẳng thức Sylvester.

 

Biết rằng đó là bất đẳng thức Sylvester nhưng thiết nghĩ cũng nên viết chứng minh ra.

 

Dùng ánh xạ tuyến tính cho đơn giản.

 

 Gọi $f,g,h: \mathbb{K}^n \rightarrow \mathbb{K}^n$ là các ánh xạ tuyến tính lần lượt nhận $A,B,C$ làm ma trận biểu diễn trong một sơ sở nào đó.

 

Khi đó, $rank A+rank B -n \le rank AB \Leftrightarrow rank f+rank g -n \le rank (f \circ g)$

 

Ta có: $rank(f \circ g)=dim (f_{|_{Im\;g}})=dim (Im \;g)-dim(Ker(f_{|_{Im\;g}}))$

 

$=rank \;g-dim(Ker(f_{|_{Im\;g}})) \ge rank \;g-dim(Ker \;f) $ vì $Ker((f_{|_{Im\;g}})) \subset Ker(f)$

 

$=rank \;g-(n-rank \;f)$

 

$\Leftrightarrow rank \;f+rank\;g -n \le rank (\;f\circ g) $

 

$\Leftrightarrow rank \;A+rank\;B-n \le rank \;AB$




#540616 Chứng minh $ det(A+B)=0$

Posted by phudinhgioihan on 13-01-2015 - 01:09 in Đại số tuyến tính, Hình học giải tích

Cho $A,B$ là các ma trận thực thỏa mãn $A.A^T=I, B.B^T=I$ với $B^T,A^T$ là các ma trận chuyển vị của $A,B$. Biết $detA\neq detB$. Chứng minh rằng $ det(A+B)=0$

 

Vì $\det A=\det A^T$ nên từ đề bài có được $\det A,\det B \in \{1; -1\}$, do $\det A \neq \det B$ nên suy ra $\det A+\det B=0$

 

$$\det A^T \det (A+B)=\det (I+A^TB)=\det (I+B^TA)=\det B^T \det(B+A)=-\det A \det(A+B)$$

 

$$\Rightarrow \det(A+B)=0$$




#540607 Về tam giác hóa ma trận

Posted by phudinhgioihan on 13-01-2015 - 00:29 in Đại số tuyến tính, Hình học giải tích

Có lẽ ý của lời giải ấy là đưa cả ba ma trận về dạng tam giác trên bằng dạng chuẩn Jordan. Chỗ sau thì từ au+bv+cw=0 ta có hệ gồm 2 phương trình liên quan đến phần thực và phần ảo hệ này luôn có nghiệm khác 0 vì hạng của hệ luôn nhỏ hơn số ẩn(có thể giải thích cách khác là không gian phức trên trường thực có chiều 2 luôn phương trình au+bv+cw=0 luôn có nghiệm khác 0). Điều anh vẫn chưa hiểu là làm sao có một cơ sở để cả 3 ma trận đều có dạng tam giác trên(có lẽ là do tính giao hoán của 3 ma trận này).

 

Ba ma trận này giao hoán từng đôi và tam giác hóa được trên $\mathbb{C}$ nên tam giác hóa được đồng thời.




#539786 Chuỗi số sau hội tụ khi nào: $$\sum_{n=1}^{+...

Posted by phudinhgioihan on 05-01-2015 - 21:38 in Giải tích

Chuỗi số sau hội tụ khi nào:
$$\sum_{n=1}^{+\infty } \sin \frac{1}{n^p}\tan\frac{1}{n^q}$$
Điều kiện  $p>0, q>0.$
 

 

Do $\lim_{n \to +\infty}\frac{1}{n^p}=\lim_{n \to +\infty}\frac{1}{n^q}=0$ nên $ \exists N_0 \in \mathbb{N}, \forall n \ge N_0, 0<\frac{1}{n^p}, \frac{1}{n^q}<\frac{\pi}{2} $

 

Suy ra, $\forall n \ge N_0, \sin \dfrac{1}{n^p}\tan\dfrac{1}{n^q}>0$

 

Chuỗi $\sum_{n=1}^{+\infty } \sin \frac{1}{n^p}\tan\frac{1}{n^q}$ và $\sum_{n=N_0}^{+\infty } \sin \frac{1}{n^p}\tan\frac{1}{n^q}$ có cùng tính chất nên ta chỉ cần tìm điều kiện $(p;q)$ để $\sum_{n=N_0}^{+\infty } \sin \frac{1}{n^p}\tan\frac{1}{n^q}$ hội tụ.

 

 

Do $\lim_{n \to +\infty} \dfrac{\sin \frac{1}{n^p}\tan\frac{1}{n^q}}{\frac{1}{n^{p+q}}}=1$ nên $\exists N_1 \in \mathbb{N},N_1>N_0 \; , \; \begin{cases}0<\sin \frac{1}{n^p}\tan\frac{1}{n^q}<\frac{2}{n^{p+q}} \\ 0<\frac{1}{n^{p+q}}<2\sin \frac{1}{n^p}\tan\frac{1}{n^q} \end{cases}$

 

Suy ra chuỗi $\sum_{n=N_0}^{+\infty } \sin \frac{1}{n^p}\tan\frac{1}{n^q}$ hội tụ khi và chỉ khi $\sum_{n=N_0} \frac{1}{n^{p+q}}$ hội tụ.

 

Chuỗi $\sum_{n=N_0} \frac{1}{n^{p+q}}$ hội tụ nếu và chỉ nếu $p+q>1$, do đó $\sum_{n=N_0}^{+\infty } \sin \frac{1}{n^p}\tan\frac{1}{n^q}$ hội tụ nếu và chỉ nếu $p+q>1$

 

Như vậy, $\sum_{n=1}^{+\infty } \sin \frac{1}{n^p}\tan\frac{1}{n^q}$ hội tụ khi và chỉ khi $p+q>1, \; (p;q) \in (\mathbb{R}_+^*)^2$.




#491640 Đề thi Olympic Toán sinh viên 2014

Posted by phudinhgioihan on 09-04-2014 - 13:16 in Thảo luận về các kì thi, các kì kiểm tra Toán sinh viên

Môn Đại số

 

Bài 1:

a) Chứng minh rằng :

 

$\det \begin{pmatrix}1 &a_1  &a_1(a_1-1)  &a_1(a_1-1)(a_1-2) \\1 &a_2  &a_2(a_2-1)  & a_2(a_2-1)(a_2-2)\\1& a_3 & a_3(a_3-1) &a_3(a_3-1)(a_3-2) \\1& a_4 &a_4(a_4-1)  &a_4(a_4-1)(a_4-2) \end{pmatrix}=\prod_{1 \le i <j \le 4}(a_j-a_i)$

b) Giả thiết $a_1, a_2, a_3, a_4$ là các số nguyên, chứng minh $\prod_{1 \le i <j \le 4} (a_j-a_i)$ chia hết cho 12.

 

Bài 2:  Cho các số thực phân biệt $a_1,a_2, a_3$. Chứng minh rằng với mọi bộ số thực $b_1, b_2, b_3$ tồn tại duy nhất một đa thức $P(x)$ bậc không quá 5 thỏa mãn: $P(a_i)=P'(a_i)=b_i, \; i=1,\; 2,\; 3$ , ở đây $P'$ ký hiệu đạo hàm của đa thức $P$.

 

Bài 3:

a) Ký hiệu $V_4$ là không gian vecto các đa thức với hệ số thực với bậc không quá 4. Định nghĩa ánh xạ $e: V_4 \rightarrow V_4$ như sau: với mỗi đa thức $f \in V_4, \; e(f)=\sum_{i=0}^4 \dfrac{f^{(i)}}{i!}$.

Chứng minh rằng $e$ là ánh xạ tuyến tính khả nghịch từ $V_4$ vào chính nó.

b) Ký hiệu $V$ là không gian vecto các đa thức hệ số thực. Với mỗi đa thức $f$, đặt $e(f)=\sum_{i=0}^{\infty} \dfrac{f^{(i)}}{i!}$. Chứng minh rằng $e$ là ánh xạ tuyến tính khả nghịch từ $V$ vào chính nó.

 

Bài 4:

a) Cho ma trận khối $X=\begin{pmatrix}E_m &B \\C&E_n\end{pmatrix}$ được tạo thành từ các ma trận đơn vị $E_m, E_n$ cấp $m,n$ tương ứng và các ma trận $B,C$ với kích thước $m \times n$ và $n \times m$ tương ứng.

Chứng minh rằng $\det(X)=\det(E_n-CB)=\det(E_m-BC)$.

b) Tổng quát, cho ma trận khối $X=\begin{pmatrix}A &B \\ C &D \end{pmatrix}$ , trong đó $A,D$ là các ma trận vuông, $A$ khả ngịch, chứng minh rằng $\det(X)=\det(A) \det(D-CA^{-1}B) $

 

Thí sinh chọn một trong hai câu của bài sau:

 

Bài 5:

a) Cho $P$ là một đa thức bậc $n$ với hệ số hữu tỷ. Giả sử số thực $a$ là nghiệm của $P$ với bội $> \frac{n}{2}$. Chứng minh rằng $a$ là một số hữu tỷ.

b) Trên hình vuông $ABCD$ ta định nghĩa đường đi giữa hai đỉnh $X, Y$ (không nhất thiết phân biệt) là một dãy các đỉnh kề nhau $XX_1X_2... X_{n-1}Y$ :  như vậy $X_0=X, .X_1,..., X_{n-1}, X_n=Y$ là các đỉnh của hình vuông và $X_iX_{i+1}$ là cạnh của hình vuông, số $n$ được gọi là độ dài của đường đi. Với mỗi số tự nhiên $n$, gọi $x_n, y_n, z_n$ tương ứng là số các đường đi độ dài $n$ giữa: một đỉnh và chính nó, một đỉnh và một đỉnh cố định kề nó, một đỉnh và đỉnh đối diện ( đỉnh đối xứng qua tâm).

Ví dụ $x_0=1, y_0=0, z_0=0, x_1=0, y_1=1, z_1=0, x_2=2, y_2=0, z_2=2 $.

1) Thiết lập công thức truy hồi cho $x_n,y_n,z_n$.

2) Tìm công thức tổng quát của $x_n,y_n,z_n$.




#469200 Cho tập $A \subset \mathbb{R}$ bị chặn, chứng m...

Posted by phudinhgioihan on 06-12-2013 - 01:45 in Giải tích

Do tập $A \subset \mathbb{R}$ và bị chặn nên ta giả sử $A=(a,b); a,b \in \mathbb{R}$ và $a<b$

 

Từ đó ta được $-A=(-b,-a)$

 

Xét $A$ có $\sup(A)=b;\inf(A)=a$

 

Xét $-A$ có $\sup(-A)=-a;\inf(-A)=-b$

 

Từ đó ta được $\sup(-A)=-\inf(A)=-a;\inf(-A)=-\sup(A)=-b$

 

Cơ sở nào để có thể quy trường hợp tổng quát về trường hợp riêng khi tập hợp chỉ có hai phần tử ? Nếu $A$ hữu hạn thì có thể giả thiết được nhưng $A$ vô hạn thì sao?




#469197 Chứng minh bất đẳng thức hình học

Posted by phudinhgioihan on 06-12-2013 - 00:43 in Hình học

Cho hai đoạn thẳng AB, CD có cùng độ dài bằng 1 và cắt nhau tại I, đồng thời $\widehat{AIC}=60^{0}$. Chứng minh rằng AC+BD >1.

 

Post từ 23-9 mà giờ anh mới biết @@

 

$$\begin{align} AC+BD &=\sqrt{IA^2+IC^2-IA.IC}+\sqrt{IB^2+ID^2-IB.ID} \\ &=\sqrt{(IA+IC)^2-3IA.IC}+\sqrt{(IB+ID)^2-3IB.ID} \\ &\ge \sqrt{(IA+IC)^2-\frac{3}{4}(IA+IC)^2}+\sqrt{(IB+ID)^2-\frac{3}{4}(IB+ID)^2} \\ &= \dfrac{IA+IC+IB+ID}{2} =1 \end{align}$$

 

Dấu bằng trong các bất đẳng thức xảy ra khi và chỉ khi $IA=IC \wedge IB=ID $ tức $\Delta IAC$ cân tại $I$.

Đề bài chứng minh $AC+BD>1$ là chưa chính xác. Lấy đơn giản trường hợp $I$ là trung điểm của cả $AB$ và $CD$ là thấy ngay $AC+BD=1$




#469196 Cho tập $A \subset \mathbb{R}$ bị chặn, chứng m...

Posted by phudinhgioihan on 06-12-2013 - 00:23 in Giải tích

Cho tập $A \subset \mathbb{R}$ bị chặn và $-A=\left \{ -x:x\in A \right \}$

CMR: $sup (-A) = -inf A$ và $inf (-A) = -sup A$

 

Bạn có thể tham khảo thêm bài viết về supremum và infimum

 

https://www.math.ucd...r/m125b/ch2.pdf

http://math.berkeley...outs/supinf.pdf




#469160 $\int \frac{x^2}{\left(x\sin x+\...

Posted by phudinhgioihan on 05-12-2013 - 22:13 in Giải tích

Tìm nguyên hàm:

 

$\int \frac{x^2}{\left(x\sin x+\cos x\right)^2}dx$

 

Nhận thấy $\left( \frac{x}{\cos x} \right)^\prime=\frac{x\sin x+\cos x}{\cos^2 x}$

$\left(x\sin x+\cos x\right)^\prime=x\cos x$ , do đó

 

$$\begin{align}\int \frac{x^2}{\left(x\sin x+\cos x\right)^2}dx &=\int \frac{x}{\cos x} \frac{x\cos x}{\left( x\sin x+\cos x \right)^2}dx \\ &=-\dfrac{x}{\cos x (x\sin x+\cos x)}+\int \dfrac{x\sin x+\cos x}{\cos^2 x} \dfrac{1}{x\sin x+\cos x}dx \\ &=\tan x-\dfrac{x}{\cos x (x\sin x+\cos x)}+constant \end{align}$$




#469158 $\int_{0}^{2\pi}\frac{cosx}...

Posted by phudinhgioihan on 05-12-2013 - 22:11 in Giải tích



Nhưng vì trong khoảng $\left ( 0,\: 2\pi \right ]$ thì $\frac{\cos x}{x}$ có dương đâu??

Do $\cos x \ge 0 \; \forall x \in [0;\frac{\pi}{2}] \cup [\frac{3\pi}{2};2\pi]$ và  $\cos x \le 0 \; \forall x \in [\frac{\pi}{2};\frac{3\pi}{2}] $ nên

 

$\int_\frac{\pi}{2}^{2\pi} \frac{\cos x}{x} dx$ có thể là hằng số âm, do đó phải có $\int_0^\frac{\pi}{2} \frac{\cos x}{x}dx =+\infty$ mới có thể chắc chắn $\int_0^{2\pi} \frac{\cos x}{x}dx >0$ được.




#469147 $\int_{0}^{2\pi}\frac{cosx}...

Posted by phudinhgioihan on 05-12-2013 - 21:55 in Giải tích

Anh chỉ cần chỉ ra $\frac{\cos x}{x}>0,\: x\in \left (0;\:\frac{\pi}{2} \right ]\to \int_{0}^{\frac{\pi}{2}}\frac{\cos x}{x}dx>0$

 

Còn $\int_{0}^{\frac{\pi}{2}}\frac{1}{x}dx=<0\left ( =-\infty \right )$ mà!!

 

$\int_0^1 \frac{dx}{x}=+\infty$ mới đúng.

 

Chỉ cần chỉ ra $\frac{\cos x}{x}>0,\: x\in \left (0;\:\frac{\pi}{2} \right ]\to \int_{0}^{\frac{\pi}{2}}\frac{\cos x}{x}dx>0$ là chưa đủ, vì nếu $\int_{0}^{\frac{\pi}{2}}\frac{\cos x}{x}dx$ là hằng số thì vẫn có thể $\int_0^{2\pi} \frac{\cos x}{x}<0$.




#469099 $\int_{0}^{2\pi}\frac{cosx}...

Posted by phudinhgioihan on 05-12-2013 - 20:46 in Giải tích

Xét dấu:

 

$\int_{0}^{2\pi}\frac{cosx}{x}dx$

 

$$\int_0^{2\pi} \dfrac{\cos x}{x}dx=\int_0^{\frac{\pi}{2}} \frac{\cos x}{x}dx+\int_\frac{\pi}{2}^{2\pi} \frac{\cos x}{x}dx $$

 

Do $\frac{\cos x}{x}>0 \; \forall x \in (0;\frac{\pi}{2}] $ cùng với $\frac{\cos x}{x} \underset{x \to 0^+}\sim \frac{1}{x} $ và $\int_0^\frac{\pi}{2} \frac{dx}{x}=+\infty$ nên $\int_0^\frac{\pi}{2} \frac{\cos x}{x}dx=+\infty$. Đồng thời $\int_\frac{\pi}{2}^{2\pi} \frac{\cos x}{x}dx$ là hằng số nên $\int_0^{2\pi} \frac{\cos x}{x}dx=+\infty$




#469060 Tính các tích phân xác định sau bằng định nghĩa

Posted by phudinhgioihan on 05-12-2013 - 19:19 in Giải tích



Tính các tích phân sau bằng định nghĩa

1)$\int_{0}^{\frac{\pi }{2}} \cos xdx$

 

Với $n \in \mathbb{N}^* $, xét phân hoạch chia đoạn $[0;\frac{\pi}{2}]$ thành $n$ đoạn bởi các điểm $x_i=\frac{\pi i}{2n} \;, i=0..n$

 

Hàm $f(x)=\cos x$ khả tích trên $[0;\frac{\pi}{2}]$ nên, $$\int_{0}^{\frac{\pi }{2}} \cos xdx=\lim_{n \to +\infty} \sum_{i=0}^{n-1} (x_{i+1}-x_i)\cos (x_{i+1}) $$

 

$$=\lim_{n \to +\infty} \frac{\pi}{2n}\sum_{i=0}^{n-1}\cos\left(\frac{\pi (i+1)}{2n}\right) \\ =\lim_{n \to +\infty}\frac{\pi}{2n}\dfrac{\sin(\frac{n\pi}{4n})\sin((n+1)\frac{\pi}{4n})}{\sin\left(\frac{\pi}{4n}\right)}=1$$




#468967 $I=\int_{0}^{\infty}\ln\left(1+...

Posted by phudinhgioihan on 05-12-2013 - 02:26 in Giải tích



Tìm điều kiện của $\alpha >0$ để tích phân hội tụ: $$I=\int_{0}^{\infty}\ln\left(1+\frac{e^{\frac{3}{x}}-1}{\alpha}\right)dx$$

 

 

 

$$I=\int_{0}^{+\infty}\ln\left(1+\frac{e^{\frac{3}{x}}-1}{\alpha}\right)dx \\ =\int_0^1 \ln\left(1+\frac{e^{\frac{3}{x}}-1}{\alpha}\right)dx+\int_1^{+\infty}\ln\left(1+\frac{e^{\frac{3}{x}}-1}{\alpha}\right)dx $$

 

Do $\ln\left(1+\frac{e^{\frac{3}{x}}-1}{\alpha}\right)dx>\ln 1=0 \;, \forall x \in (0;1]$ nên $\int_0^1 \ln\left(1+\frac{e^{\frac{3}{x}}-1}{\alpha}\right)dx >0 $

 

Ta có $\ln\left(1+\frac{e^{\frac{3}{x}}-1}{\alpha}\right)>\ln 1=0 \;, \forall x \ge 1 $ và

 

$$\ln\left(1+\frac{e^{\frac{3}{x}}-1}{\alpha}\right) \underset{x \to +\infty}{\sim} \dfrac{e^\frac{3}{x}-1}{\alpha} \underset{x \to +\infty}{\sim} \dfrac{3}{\alpha x} $$

 

Tích phân $\int_1^{+\infty} \dfrac{3}{\alpha x}dx =+\infty \;, \forall \alpha>0$ nên $\int_1^{+\infty} \ln\left(1+\frac{e^{\frac{3}{x}}-1}{\alpha}\right)dx=+\infty \;, \forall \alpha>0$

 

Tóm lại, $\forall \alpha>0$ thì $I=+\infty$




#468965 $f(x)= \left\{\begin{matrix}\frac...

Posted by phudinhgioihan on 05-12-2013 - 01:04 in Giải tích

 

Cho hàm số $$f(x)=  \left\{\begin{matrix}\frac{3-x^{2}}{2} &(0 \le x \le 1)  & \\ \frac{1}{x}& (1<x<+\infty )) & \end{matrix}\right.$$

Chứng minh hàm f(x)thoảmãn các điều kiện của định lý Lagrange trên đoạn [0, 2], 

tìm các giá trịtrung gian c trong công thức sốgia hưu hạn trong trườn hợp này.
 
Em không biết  tìm giá trị trung gian thế nào, Theo đề bài là các giá trị trung gian. Em định chia ra 2 khoảng [0,1] và [1,2] do ở đây có 2 đạo hàm rồi theo công thức số gia hữu hạn tìm c.Tuy nhiên nếu theo cách lập luân đó thì em có thể chia ra rất nhiều khoảng và ra rất nhiều c nên em không biết làm sao nữa.
Nhờ các thầy cô giải giúp em ạ.Em cảm ơn

 

 

Hiển nhiên $f$ khả vi trên $[0;1]$ và $(1;+\infty) $. Tại $x=1$

 

$$\lim_{x \to 1^-} \dfrac{f(x)-f(1)}{x-1}=\lim_{x \to 1^-}\dfrac{\frac{3-x^2}{2}-1}{x-1}=-1$$

 

$$\lim_{x \to 1^+} \dfrac{f(x)-f(1)}{x-1}=\lim_{x \to 1^+} \dfrac{\frac{1}{x}-1}{x-1}=-1$$

 

Suy ra $\lim_{x \to 1^-}\dfrac{f(x)-f(1)}{x-1}=\lim_{x \to 1^+} \dfrac{f(x)-f(1)}{x-1}$ nên $f'(1)$ tồn tại và $f'(1)=-1$

 

Như vậy $f$ khả vi trên $[0;+\infty)$ do đó cũng khả vi trên $[0;2]$ tức thoả điều kiện của định lý Lagrange.

 

$$f'(x)= \left\{\begin{matrix}-x & 0 \le x \le 1 \\ -\frac{1}{x^2} & x>1 \end{matrix}\right. $$

 

$f(2)-f(0)=-1=(2-0)\frac{-1}{2}=(2-0)f'(\frac{1}{2})$

 

Tức giá trị trung gian cần tìm trong công thức số gia hữu hạn là $\frac{1}{2}$




#466399 Nanyang Technological University Mathematics Competition

Posted by phudinhgioihan on 24-11-2013 - 01:50 in Thảo luận về các kì thi, các kì kiểm tra Toán sinh viên

Đại học công nghệ Nanyang (NTU) là một trong những đại học danh tiếng của khu vực Đông Nam Á, hiện tại có rất nhiều sinh viên Việt Nam đang theo học tại đây và có cả giảng viên là người Việt Nam nữa :D (qua đó học đồng hương hơi bị đông ^_^).

Đây là đề thi của ĐH Nanyang nhằm khuyến khích phong trào Toán và hơn hết là chọn đội tuyển tham dự IMC kể từ lần đầu tiên năm 2007 được mình tổng hợp lại đầy đủ, có cả đề thi chọn đội tuyển IMC nữa :)

 

P/s: đề có vài câu dân VN rất quen thuộc :D

Attached Files




#465971 The William Lowell Putnam Mathematical Competition 1938-2008

Posted by phudinhgioihan on 22-11-2013 - 17:22 in Thảo luận về các kì thi, các kì kiểm tra Toán sinh viên

The William Lowell Putnam Mathematical Competition, often abbreviated to the Putnam Competition, is an annual mathematics competition for undergraduate college students enrolled at institutions of higher learning in the United States and Canada (regardless of the students' nationalities). It awards scholarships with cash prizes ranging from $250 to $2,500 for the top students and $5,000 to $25,000 for the top schools, plus the top ten individual scores get tuition waived at Harvard, and the top 100 individual scores have their names mentioned by rank to leading universities. It is considered by many to be the most prestigious university-level mathematics examination in the world. The competition was founded in 1927 by Elizabeth Lowell Putnam in memory of her husband William Lowell Putnam, who was an advocate of intercollegiate intellectual competition. The exam has been offered annually since 1938 and is administered by the Mathematical Association of America.

 

 

Putnam 1938-1964

 

Putnam 1965-1984

 

Putnam 1985-2000

 

Putnam 2001-2008




#465966 Miklós Schweizer Competitions 1962-1991

Posted by phudinhgioihan on 22-11-2013 - 17:05 in Thảo luận về các kì thi, các kì kiểm tra Toán sinh viên

The Miklós Schweitzer Competition (Schweitzer Miklós Matematikai Emlékverseny in Hungarian) is an annual Hungarian mathematics competition for university undergraduates, established in 1949.

It is named after Miklós Schweitzer (1 Febr. 1923 - 28 January 1945) a young Hungarian mathematician, who died under the Siege of Budapest in the Second World War.

The Schweitzer contest is uniquely high-level among mathematics competitions. The problems, written by prominent Hungarian mathematicians, are challenging and require in-depth knowledge of the fields represented. The competition is open-book and competitors are allowed ten days to come up with solutions.

The problems on the competition can be classified roughly in the following categories: 1. Algebra 2. Combinatorics 3. Theory of Functions 4. Geometry 5. Measure Theory 6. Number Theory 7. Operators 8. Probability Theory 9. Sequences and Series 10. Topology 11. Set Theory

 

------wikipedia-----

 

Download